Difference between revisions of "2021 AMC 10B Problems/Problem 17"

m
(Undo revision 164714 by Puffer13 (talk) We wrote this solution COLLECTIVELY. So, credits should be given to all contributors.)
(Tag: Undo)
 
(20 intermediate revisions by 4 users not shown)
Line 1: Line 1:
 
==Problem==
 
==Problem==
  
Ravon, Oscar, Aditi, Tyrone, and Kim play a card game. Each person is given 2 cards out of a set of 10 cards numbered <math>1,2,3, \dots,10.</math> The score of a player is the sum of the numbers of their cards. The scores of the players are as follows: Ravon--11, Oscar--4, Aditi--7, Tyrone--16, Kim--17. Which of the following statements is true?
+
Ravon, Oscar, Aditi, Tyrone, and Kim play a card game. Each person is given <math>2</math> cards out of a set of <math>10</math> cards numbered <math>1,2,3, \dots,10.</math> The score of a player is the sum of the numbers of their cards. The scores of the players are as follows: Ravon--<math>11,</math> Oscar--<math>4,</math> Aditi--<math>7,</math> Tyrone--<math>16,</math> Kim--<math>17.</math> Which of the following statements is true?
  
<math> \textbf{(A) \ }\text{Ravon was given card 3.} \qquad \textbf{(B) \ }\text{Aditi was given card 3.} \qquad \textbf{(C) \ }\text{Ravon was given card 4.} \qquad \textbf{(D) \ }\text{Aditi was given card 4} \qquad </math> <math> \textbf{(E) \ }\text{Tyrone was given card 7}  </math>
+
<math>\textbf{(A) }\text{Ravon was given card 3.}</math>
  
==Solution==
+
<math>\textbf{(B) }\text{Aditi was given card 3.}</math>
 +
 
 +
<math>\textbf{(C) }\text{Ravon was given card 4.}</math>
 +
 
 +
<math>\textbf{(D) }\text{Aditi was given card 4.}</math>
  
Oscar must be given 3 and 1, so we rule out <math>\textbf{(A) \ }</math> and <math>\textbf{(B) \ }</math>. If Tyrone had card 7, then he would also have card 9, and then Kim must have 10 and 7 so we rule out <math>\textbf{(E) \ }</math>. If Aditi was given card 4, then she would have card 3, which Oscar already had. So the answer is <math>\boxed{ \textbf{(C) \ }\text{Ravon was given card 4.}}</math>
+
<math>\textbf{(E) }\text{Tyrone was given card 7.}</math>
  
~smarty101 and smartypantsno_3
+
==Solution==
 +
By logical deduction, we consider the scores from lowest to highest:
 +
<cmath>\begin{align*}
 +
\text{Oscar's score is 4.} &\implies \text{Oscar is given cards 1 and 3.} \\
 +
&\implies \text{Aditi is given cards 2 and 5.} \\
 +
&\implies \text{Ravon is given cards 4 and 7.} && (\bigstar) \\
 +
&\implies \text{Tyrone is given cards 6 and 10.} \\
 +
&\implies \text{Kim is given cards 8 and 9.}
 +
\end{align*}</cmath>
 +
Therefore, the answer is <math>\boxed{\textbf{(C) }\text{Ravon was given card 4.}}</math>
  
==Solution 2==
+
Certainly, if we read the answer choices sooner, then we can stop at <math>(\bigstar)</math> and pick <math>\textbf{(C)}.</math>
Oscar must be given 3 and 1. Aditi cannot be given 3 or 1, so she must have 2 and 5. Similarly, Ravon cannot be given 1, 2, 3, or 5, so he must have 4 and 7, and the answer is <math>\boxed{ \textbf{(C) \ }\text{Ravon was given card 4.}}</math>.
 
  
-SmileKat32
+
~smarty101 ~smartypantsno_3 ~SmileKat32 ~MRENTHUSIASM
  
== Video Solution by OmegaLearn (Using logical deduction) ==
+
== Video Solution by OmegaLearn (Using Logical Deduction) ==
 
https://youtu.be/zO0EuKPXuT0
 
https://youtu.be/zO0EuKPXuT0
  
Line 25: Line 37:
  
 
~IceMatrix
 
~IceMatrix
 +
==Video Solution by Interstigation==
 +
https://youtu.be/8BPKs24eyes
  
 +
~Interstigation
  
 +
==See Also==
 
{{AMC10 box|year=2021|ab=B|num-b=16|num-a=18}}
 
{{AMC10 box|year=2021|ab=B|num-b=16|num-a=18}}
 +
{{MAA Notice}}

Latest revision as of 16:18, 6 November 2021

Problem

Ravon, Oscar, Aditi, Tyrone, and Kim play a card game. Each person is given $2$ cards out of a set of $10$ cards numbered $1,2,3, \dots,10.$ The score of a player is the sum of the numbers of their cards. The scores of the players are as follows: Ravon--$11,$ Oscar--$4,$ Aditi--$7,$ Tyrone--$16,$ Kim--$17.$ Which of the following statements is true?

$\textbf{(A) }\text{Ravon was given card 3.}$

$\textbf{(B) }\text{Aditi was given card 3.}$

$\textbf{(C) }\text{Ravon was given card 4.}$

$\textbf{(D) }\text{Aditi was given card 4.}$

$\textbf{(E) }\text{Tyrone was given card 7.}$

Solution

By logical deduction, we consider the scores from lowest to highest: \begin{align*} \text{Oscar's score is 4.} &\implies \text{Oscar is given cards 1 and 3.} \\ &\implies \text{Aditi is given cards 2 and 5.} \\ &\implies \text{Ravon is given cards 4 and 7.} && (\bigstar) \\ &\implies \text{Tyrone is given cards 6 and 10.} \\ &\implies \text{Kim is given cards 8 and 9.} \end{align*} Therefore, the answer is $\boxed{\textbf{(C) }\text{Ravon was given card 4.}}$

Certainly, if we read the answer choices sooner, then we can stop at $(\bigstar)$ and pick $\textbf{(C)}.$

~smarty101 ~smartypantsno_3 ~SmileKat32 ~MRENTHUSIASM

Video Solution by OmegaLearn (Using Logical Deduction)

https://youtu.be/zO0EuKPXuT0

~ pi_is_3.14

Video Solution by TheBeautyofMath

https://youtu.be/FV9AnyERgJQ?t=284

~IceMatrix

Video Solution by Interstigation

https://youtu.be/8BPKs24eyes

~Interstigation

See Also

2021 AMC 10B (ProblemsAnswer KeyResources)
Preceded by
Problem 16
Followed by
Problem 18
1 2 3 4 5 6 7 8 9 10 11 12 13 14 15 16 17 18 19 20 21 22 23 24 25
All AMC 10 Problems and Solutions

The problems on this page are copyrighted by the Mathematical Association of America's American Mathematics Competitions. AMC logo.png